Jump to content

Lorenz Force thrust


DandelionTheory

Recommended Posts

12 minutes ago, swansont said:

Berating people for asking you to clarify makes things worse.

Your teaching methods need work.

I gave you calculations where I showed force on the center wire from the parallel sides will cancel. Force on the top section does not. 

I go on later to state if the voltage was in the 30kv range and the air gap between the bottom parts of the circuit was 1 inch, the discharge would complete the circuit while losing some momentum outside the wire returning to ground as it finishes the arc. This is an assumption, but you wanted it to make a circuit.

Edited by DandelionTheory
Link to comment
Share on other sites

2 minutes ago, DandelionTheory said:

Your teaching methods need work.

I don't think I was "teaching"

You seem to be in lecture mode, not student mode (if it was the latter you would be asking questions, rather than insisting that certain things are true)

2 minutes ago, DandelionTheory said:

I gave you calculations where I showed force on the center wire from the parallel sides will cancel. Force on the top section does not. 

You ignored part of the circuit. Of course you will get the wrong answer.

 

2 minutes ago, DandelionTheory said:

I go on later to state if the voltage was in the 30kv range and the air gap between the bottom parts of the circuit was 1 inch, the discharge would complete the circuit while losing some momentum outside the wire returning to ground as it finishes the arc. This is an assumption, but you wanted it to make a circuit.

Without a circuit, there is no current. Why would you assume there is no force on an arc?

Link to comment
Share on other sites

13 minutes ago, swansont said:

You ignored part of the circuit. Of course you will get the wrong answer

Is that before or after you understood it? The circuit is complete. I said it in the op. Attempting to make me insecure about what I've covered won't work.

15 minutes ago, swansont said:

Without a circuit, there is no current. Why would you assume there is no force on an arc?

Never said there was no force on the arc, I said the ions would be outside the wire so it wouldn't matter to the total force on the structure. Due to them being not attached.

Link to comment
Share on other sites

40 minutes ago, DandelionTheory said:

Is that before or after you understood it? The circuit is complete. I said it in the op. Attempting to make me insecure about what I've covered won't work.

You don't show a current flowing  from D to C and say "IF, the circuit were to be closed with a wire from D to C, the  total force on DC would cancel out the total force on AB." which means there isn't a closed circuit.

Your OP talks of a hypotenuse, so you did not make clear any connection between it and the diagram with thr rectangles (which are not right triangles.)

You need to disabuse yourself of the notion that your posts — which you've admitted to conceiving while you are high — contain coherent explanations. 

 
Quote

Never said there was no force on the arc, I said the ions would be outside the wire so it wouldn't matter to the total force on the structure. Due to them being not attached.

You have not supported this assertion with any physics. Why would there be no force on the structure? The electromagnetic forces do not require any mechanical attachment.

Link to comment
Share on other sites

From the title of this do I understand you are trying to design/build an electromechanical device capable of providing sustained motion of an object with mass and yet be part of that object?
Rather like an internal combustion engine or an electric motor in a car which moves along with the car?

 

Link to comment
Share on other sites

7 minutes ago, studiot said:

From the title of this do I understand you are trying to design/build an electromechanical device capable of providing sustained motion of an object with mass and yet be part of that object?
Rather like an internal combustion engine or an electric motor in a car which moves along with the car?

 

Correct. Mass is ejected in the process I mentioned in the op. I attempted to show the completed circuit but y'all needed more clarification. So I broke it down. I'm sorry it does look like I'm trying to teach you something, but it seems like I'm just not understood the first time so I've been attempted to clarify.

The completed circuit experiences force from the magnetic fields of the current in it. If some of the current is not attached to the wire loop, that momentum is not added to the total momentum of the wires.

24 minutes ago, swansont said:

Why would there be no force on the structure? The electromagnetic forces do not require any mechanical attachment.

Agreed, but the ions are not attached to a structure during the arc.

25 minutes ago, swansont said:

coherent explanations

I tried and what I got back was "wrong" with no math or anything substantial to add but an opinion. I'm trying to explain. Some credit for the things I do get would help and not add to the discontent btw.

Link to comment
Share on other sites

6 minutes ago, DandelionTheory said:

Correct. Mass is ejected in the process I mentioned in the op. I attempted to show the completed circuit but y'all needed more clarification.

I don't see any mention of mass in the opening post.

Where does this mass come from?

Are you proposing some kind of rocket or jet engine?

Link to comment
Share on other sites

15 minutes ago, studiot said:

I don't see any mention of mass in the opening post.

Where does this mass come from?

Are you proposing some kind of rocket or jet engine?

Yes, more of an add-on to an idea of electron ejection. Ions are mass. More specifically current has mass.

 

Edited by DandelionTheory
Link to comment
Share on other sites

13 minutes ago, Mordred said:

All forms of energy has a mass term that doesn't address Studiots question.

Quote

If the hypotenuse of a right triangle is open for charge carriers to be free, force applied to the system from the hypotenuse can be negated due to it not being physically attached.

My phrasing is wrong. I meant if the hypotenuse was attached it would add to the total force on the structure away from the center. Since it is not attached it still creates a magnetic field and still affects the rest of the "triangle"

When current leaves the wires it cannot add to the momentum of them internally, just externally. It's a good thing.

Edited by DandelionTheory
Link to comment
Share on other sites

27 minutes ago, DandelionTheory said:

Yes, more of an add-on to an idea of electron ejection. Ions are mass. More specifically current has mass.

You would be the second poster recently to try to create an electromechanical device without paying proper attention to the mechanical part.

In particular please explain how the 'thrust' is transmitted to the frame or body of the vehicle. ?

Link to comment
Share on other sites

18 minutes ago, studiot said:

You would be the second poster recently to try to create an electromechanical device without paying proper attention to the mechanical part.

In particular please explain how the 'thrust' is transmitted to the frame or body of the vehicle. ?

https://physics.stackexchange.com/questions/378545/how-can-laplace-lorentz-force-move-objects-and-not-charges

Link to comment
Share on other sites

22 minutes ago, DandelionTheory said:

I didn't ask how the thrust was generated (actually I did but you didn't answer that either)

I asked specifically how this is transmitted to the body of the moving body?

You know the sort of thing in the song

your leg bone is connected to your thigh bone
and your thigh bone is connected to..

etc

etc

 

Purely mechanical stuff.

Edited by studiot
Link to comment
Share on other sites

10 minutes ago, studiot said:

I didn't ask how the thrust was generated (actually I did but you didn't answer that either)

I asked specifically how this is transmitted to the body of the moving body?

You know the sort of thing in the song

your leg bone is connected to your thigh bone
and your thigh bone is connected to..

etc

etc

 

Purely mechanical stuff.

Looks like drag. To be clear that's off the top of my head, I'll take a look at the laplas force's proofs a bit more.

So the electron is connected to the atom and any charge carrier experiences force in a magnetic field, not it's own of course. So as the electron runs through the wire it takes a turn and hits space where some other electron's (from the same current) magnetic field intersects it's path, it will experience the lorentz force and the other electron will experience an opposite force. They both will drag on the wire. While the electron is outside the wire in the arc like mentioned before, no drag on the wire is calculated and therefore the laplas force does not apply to the structure but the lorentz force always applies to the electrons in or out of a wire.

Link to comment
Share on other sites

2 hours ago, DandelionTheory said:

Correct. Mass is ejected in the process I mentioned in the op. I attempted to show the completed circuit but y'all needed more clarification. So I broke it down. I'm sorry it does look like I'm trying to teach you something, but it seems like I'm just not understood the first time so I've been attempted to clarify.

The completed circuit experiences force from the magnetic fields of the current in it. If some of the current is not attached to the wire loop, that momentum is not added to the total momentum of the wires.

Why not? How does a charge carrier get ejected fro the wire without experiencing a force, and why will that not exert a reaction force on the wire? And so so on, because all the charge carriers electromagnetically interact with each other.

It seems like you are just assuming they don't, and then coming up with a solution that violates conservation of momentum. Those who have experience solving such problems know that an apparent violation of a physical law often points back to a faulty assumption.

 

2 hours ago, DandelionTheory said:

Agreed, but the ions are not attached to a structure during the arc.

You just agreed that attachment isn't necessary.

 

 

1 hour ago, DandelionTheory said:

Yes, more of an add-on to an idea of electron ejection. Ions are mass. More specifically current has mass.

 

An ion current will have mass and momentum and you need to account for it in your analysis.

Link to comment
Share on other sites

18 minutes ago, DandelionTheory said:

Looks like drag. To be clear that's off the top of my head, I'll take a look at the laplas force's proofs a bit more.

So the electron is connected to the atom and any charge carrier experiences force in a magnetic field, not it's own of course. So as the electron runs through the wire it takes a turn and hits space where some other electron's (from the same current) magnetic field intersects it's path, it will experience the lorentz force and the other electron will experience an opposite force. They both will drag on the wire. While the electron is outside the wire in the arc like mentioned before, no drag on the wire is calculated and therefore the laplas force does not apply to the structure but the lorentz force always applies to the electrons in or out of a wire.

For the third and last time,

You have a pair of wires  in a T shape dangling unsupported in space and apparently unconnected to a source of electricity.

There appears to be a magnetic field magically confined to two triangular areas also from some disembodied source unconnected to anything else.

How does this arrangement drive a (motor) car?

That is how does it push the car along the road?

Link to comment
Share on other sites

4 hours ago, DandelionTheory said:

Yes, more of an add-on to an idea of electron ejection. Ions are mass. More specifically current has mass.

Are the electrons "permanently" ejected from the system or are they circulating in the system?

Thought experiment, in addition to @studiot's queries. Your device is in a closed, blue box (picture below). The box has low mass to not interfere with the device's capacity. "The Device" means everything that is required for it to operate. Power sources, cables etc. There is no connection to the outside of the box; no electro magnetics, cables, noting. The device is started (via a timer since it is not readable from outside the box). If the device works as you intend it to do, will the box rise into the air, pushed by your device inside it? 

image.png.b5b29918a91154a8f6bf1c477dc8c70a.png

Link to comment
Share on other sites

7 hours ago, Ghideon said:

Are the electrons "permanently" ejected from the system or are they circulating in the system?

Thought experiment, in addition to @studiot's queries. Your device is in a closed, blue box (picture below). The box has low mass to not interfere with the device's capacity. "The Device" means everything that is required for it to operate. Power sources, cables etc. There is no connection to the outside of the box; no electro magnetics, cables, noting. The device is started (via a timer since it is not readable from outside the box). If the device works as you intend it to do, will the box rise into the air, pushed by your device inside it? 

image.png.b5b29918a91154a8f6bf1c477dc8c70a.png

Never said it worked in a box.

Again you refuse to read what was stated and assume im making something else. 

I've said it works by ejecting mass and laplas force's on the structure. I said momentum is lost when the arc is not attached to wires. Take a second and comprehend the idea child. I said it works in a vacuum. Not a box.

9 hours ago, studiot said:

You have a pair of wires  in a T shape dangling unsupported in space and apparently unconnected to a source of electricity.

There appears to be a magnetic field magically confined to two triangular areas also from some disembodied source unconnected to anything else.

How does this arrangement drive a (motor) car?

That is how does it push the car along the road?

Back up. We just had an issue with being on the same page. So you follow?

 

Do I have some credit or are you attempting to "find some magic" I don't intend to claim? Can I be seen as someone trying

I told you how it puts force on the wires, I did not tell you it made enough thrust to push a car. I never had claims it did anything else. I said it was a circuit, I said it had wires, I said there was an arc to complete the circuit, I said the lorentz force calculates the total force on the current and the laplas force calculates the force on the wires. Do I need to tell you current creates a magnetic field around the wire... To which the magnetic fields I am calculating are originated; or can you grasp that?

Link to comment
Share on other sites

12 hours ago, swansont said:

Why not? How does a charge carrier get ejected fro the wire without experiencing a force, and why will that not exert a reaction force on the wire? And so so on, because all the charge carriers electromagnetically interact with each other.

Now you're getting it. It does, the wire goes in one direction and the electron in the other. All charge carriers experience the lorentz force, only ones in the wire contribute to drag in the laplas force on the wires. The free electron in the arc still does work on the other electrons in the wire and vice versa, the free electron in the arc just does not contribute to the drag part of the laplas force calculation on the structure.

Link to comment
Share on other sites

6 hours ago, DandelionTheory said:

Never said it worked in a box.

Ok! The box is a thought experiment intended to help with the discussion. Not a claim about the device. 

6 hours ago, DandelionTheory said:

Again you refuse to read what was stated and assume im making something else. 

I have read what you state but I have trouble understanding some of it. So I ask for clarifications.

6 hours ago, DandelionTheory said:

I've said it works by ejecting mass and laplas force's on the structure. I said momentum is lost when the arc is not attached to wires.

 Your drawing does not show ejection of mass. It shows currents that seem to be circulating in the system. Where is mass ejected, never to be returned to the system? Momentum of ejected mass will generate thrust. If mass is returned or kept in the system momentum conservation says there is no thrust on the system.

Link to comment
Share on other sites

7 hours ago, DandelionTheory said:

Now you're getting it. It does, the wire goes in one direction and the electron in the other. All charge carriers experience the lorentz force, only ones in the wire contribute to drag in the laplas force on the wires. The free electron in the arc still does work on the other electrons in the wire and vice versa, the free electron in the arc just does not contribute to the drag part of the laplas force calculation on the structure.

What is the “laplas” force? Do you mean Laplace?

The electrons can’t do work without contributing to the force. In any event, you’re just asserting that they don’t contribute. You haven’t done any analysis to show it.

Link to comment
Share on other sites

33 minutes ago, swansont said:

What is the “laplas” force? Do you mean Laplace?

The electrons can’t do work without contributing to the force. In any event, you’re just asserting that they don’t contribute. You haven’t done any analysis to show it.

You're correct I misspelled it. 

I'm asserting the electron will not contribute to the Laplace force when it is not in a wire.

Link to comment
Share on other sites

1 hour ago, swansont said:

Show your analysis. How do they not? 

The difference between the Laplace force and the Lorentz force is the electron is physically in the wire with the Laplace force and does drag on the wire where with the lorentz force the charge carrier is free and is a calculation on the charge carrier itself no wire. 

If an electron is not in a wire it cannot contribute to the drag on the wire in the Laplace calculation. Once it is out of the wire it's a free charge carrier susceptible to all normal forces with no drag on the wire.

 

Because it's not in the wire.

Edited by DandelionTheory
Link to comment
Share on other sites

1 hour ago, DandelionTheory said:

The difference between the Laplace force and the Lorentz force is the electron is physically in the wire with the Laplace force and does drag on the wire where with the lorentz force the charge carrier is free and is a calculation on the charge carrier itself no wire. 

If an electron is not in a wire it cannot contribute to the drag on the wire in the Laplace calculation. Once it is out of the wire it's a free charge carrier susceptible to all normal forces with no drag on the wire.


 

Describing the equations is not the same as solving them. I strongly suspect you would not do so correctly, seeing as you insist on an incorrect answer.

 

Quote

Because it's not in the wire.

And electromagnetic forces on the wire disappear when charges aren’t in the wire?  
 

Hint 1: is the wire neutral, or does it have a net charge?

Hint 2: what is the path of the arc?

Link to comment
Share on other sites

Create an account or sign in to comment

You need to be a member in order to leave a comment

Create an account

Sign up for a new account in our community. It's easy!

Register a new account

Sign in

Already have an account? Sign in here.

Sign In Now
×
×
  • Create New...

Important Information

We have placed cookies on your device to help make this website better. You can adjust your cookie settings, otherwise we'll assume you're okay to continue.